(Luận văn) bất đẳng thức trong số học và một số dạng toán liên quan

80 2 0
(Luận văn) bất đẳng thức trong số học và một số dạng toán liên quan

Đang tải... (xem toàn văn)

Tài liệu hạn chế xem trước, để xem đầy đủ mời bạn chọn Tải xuống

Thông tin tài liệu

ĐẠI HỌC THÁI NGUYÊN TRƯỜNG ĐẠI HỌC KHOA HỌC LÊ THỊ HỒNG THÚY lu an n va p ie gh tn to BẤT ĐẲNG THỨC TRONG SỐ HỌC VÀ MỘT SỐ DẠNG TOÁN LIÊN QUAN nl w d oa LUẬN VĂN THẠC SĨ TOÁN HỌC nf va an lu z at nh oi lm ul z m co l gm @ an Lu THÁI NGUYÊN - 2018 n va ac th si ĐẠI HỌC THÁI NGUYÊN TRƯỜNG ĐẠI HỌC KHOA HỌC LÊ THỊ HỒNG THÚY lu an n va p ie gh tn to BẤT ĐẲNG THỨC TRONG SỐ HỌC VÀ MỘT SỐ DẠNG TOÁN LIÊN QUAN d oa nl w Mã số: 46 01 13 nf va an lu Chuyên ngành: PHƯƠNG PHÁP TOÁN SƠ CẤP lm ul LUẬN VĂN THẠC SĨ TOÁN HỌC z at nh oi z Người hướng dẫn khoa học: GS.TSKH Nguyễn Văn Mậu m co l gm @ an Lu Thái Nguyên - 2018 n va ac th si i Mục lục MỞ ĐẦU ii lu Chương Các tính tốn tập hữu hạn số ngun an n va Số nguyên tính chất liên quan 1.2 Một số đồng thức số học 1.2.1 Một số đẳng thức hàm d(n), σ(n) ϕ(n) 1.2.2 Đẳng thức tổng bình phương 10 1.2.3 Biểu diễn số tự nhiên thành tổng lập phương 15 p ie gh tn to 1.1 28 nl w Chương Bất đẳng thức số học Bất đẳng thức tập số nguyên 28 2.2 Bất đẳng thức lớp hàm số học 32 d oa 2.1 an lu 60 nf va Chương Một số dạng toán liên quan Các dạng toán bất đẳng thức số học qua kỳ Olympic 3.2 Các đề toán toán rời rạc liên quan 64 z at nh oi lm ul 3.1 60 3.2.1 Một số toán cực trị tập số nguyên 3.2.2 Một số toán sử dụng phương pháp suy luận 68 74 gm @ 75 m co l TÀI LIỆU THAM KHẢO z KẾT LUẬN 64 an Lu n va ac th si ii MỞ ĐẦU Chuyên đề số học nội dung quan trọng bậc trung học phổ thông Các dạng toán đếm số phần tử, so sánh thứ tự số tập hợp nội dung đề thi HSG quốc gia Olympic toán khu vực lu an quốc tế n va Đặc biệt lý thuyết số, hàm số học liên quan đến tính tốn tn to ước số nguyên, gắn với phép đếm số ước số dạng toán liên quan đến biểu diễn số nguyên trọng tâm khảo sát đẳng thức gh p ie bất đẳng thức số học Luận văn nhằm mục đích tìm hiểu chi tiết tính chất hàm số học nl w số dạng toán bất đẳng thức cực trị liên quan số học d oa Ngoài phần Mở đầu Kết luận, luận văn chia thành ba chương đề cập đến vấn đề sau đây: lu nf va an Chương trình bày tốn đếm, ước lượng thứ tự Chương trình bày dạng bất đẳng thức tính tốn liên quan đến lm ul tập rời rạc hàm số học z at nh oi Chương trình bày số toán cực trị đề thi học sinh giỏi quốc gia, Olympic khu vực quốc tế liên quan đến bất đẳng thức số học Luận văn hoàn thành hướng dẫn khoa học Nhà giáo nhân z dân, GS.TSKH Nguyễn Văn Mậu Tác giả xin bày tỏ lòng biết ơn chân @ gm thành sâu sắc tới GS - Người thầy nghiêm khắc, tận tâm công việc l truyền thụ nhiều kiến thức quý báu kinh nghiệm nghiên cứu m co khoa học cho tác giả suốt trình học tập, nghiên cứu đề tài Tác giả xin bày tỏ lòng biết ơn chân thành đến Ban Giám hiệu, Phòng an Lu đào tạo sau đại học, khoa Toán - Tin trường Đại học Khoa học - Đại học n va ac th si iii Thái Nguyên, thầy cô giáo tham giảng dạy hướng dẫn khoa học cho lớp Cao học toán K10C Tác giả xin chân thành cảm ơn Ban Giám hiệu, tập thể giáo viên tốn trường THPT Lý Nhân Tơng, thành phố Bắc Ninh gia đình tạo điều kiện cho tác giả có hội học tập nghiên cứu lu an n va p ie gh tn to d oa nl w nf va an lu z at nh oi lm ul z m co l gm @ an Lu n va ac th si Chương Các tính tốn tập hữu hạn số ngun lu an 1.1 Số nguyên tính chất liên quan va n Trước tiên, ta xét số hàm số học tn to Định nghĩa 1.1 (Hàm số Euler ϕ(n)) Cho số tự nhiên n ≥ Ta ký hiệu ϕ(n) gh p ie số số tự nhiên bé n nguyên tố với n Quy ước ϕ(1) = w Định lý 1.1 Hàm ϕ(n) có tính chất nhân tính theo nghĩa: Nếu a, b hai số oa nl nguyên tố d ϕ(ab) = ϕ(a)ϕ(b) an lu Chứng minh nf va Rõ ràng ta giải thiết a > 1, b > Các số nguyên dương không vượt a+2 2a + 2a + ka + ka + (b − 1)a + (b − 1)a + 2a 3a (k + 1)a ba l gm a+1 a @ z z at nh oi lm ul ab liệt kê sau: Các số thành bảng có dạng ax + y , ≤ x ≤ b − 1, ≤ y ≤ a co m Xét số cột thứ y Ta có (ax + y, a) = (y, a) Vì số nguyên tố với ab an Lu nguyên tố với a b, số phải nằm cột thứ y mà (y, a) = Có thảy ϕ(a) cột Xét cột thứ y , với (y, a) = n va ac th si Các số cột y, a + y, 2a + y, , (b − 1)a + y Giả sử rx số dư chia ax + y cho b Như (ax + y, b) = (rx , b) Dễ dàng thấy (a, b) = nên rx1 6= rx2 với x1 6= x2 Như ta có đẳng thức tập hợp {r0 , r1 , , rb−1 } = {0, 1, , b − 1} Vậy số x mà (ax + y, b) = số x mà (rx , b) = tức ϕ(b) Vậy có ϕ(a)ϕ(b) số nguyên tố với a ngun tố với b Đó lu số nguyên tố với ab Nói cách khác ϕ(ab) = ϕ(a)ϕ(b) an Từ định lý ta suy cơng thức tính ϕ(n) sau n va tn to Định lý 1.2 Giả sử n = pα1 pαk k phân tích tiêu chuẩn n > Khi      1 ϕ(n) = n − 1− − p2 pk ie gh p1 p Chứng minh nl w Theo định lý 1.1, ta có ϕ(n) = ϕ(pα1 ) ϕ(pαk k ) oa Định lý chứng minh ta chứng tỏ ứng với p số nguyên d tố ϕ(pα ) = pα (1 − ) Thật vậy, p nguyên tố nên với k ≤ pα p (k, p) = k p h pα i α Số số k ≤ p bội p = pα−1 nf va an lu lm ul Vậy p z at nh oi ϕ(pα ) = pα − pα−1 = pα (1 − ) p ϕ(360) = 360 −  1−  = 96 gm @ 1− z Ví dụ 1.1 Với n = 360 = 23 32   Tầm quan trọng hàm ϕ(n) số học thể định lý Euler m co l Sau suy rộng định lý Euler Định lý 1.3 (Định lý Euler mở rộng) Cho a m hai số tự nhiên Khi (mod m) n va am ≡ am−ϕ(m) an Lu ta có ac th si Chứng minh Ta phải chứng minh A = am − am−ϕ(m) = am−ϕ(m) (aϕ(m) − 1) chia hết cho m Giả sử m có phân tích tiêu chuẩn m = q1α1 q2α2 qkαk Ta chứng minh (a, qi ) = (aϕ(m) − 1) q αi , cịn a q am−ϕ(m) q αi , A m lu an Thật vậy, (a, qi ) = theo định lý Euler va αi n (aϕ(qi ) − 1) qiαi gh tn to Mặt khác, p ie ϕ(qiαi ) = qiαi (1 − ) qi ước ϕ(m) (suy từ cơng thức tính ϕ(m)) nl w Do oa d ) − 1) q αi nf va an lu Nếu a qi αi (aϕ(m) − 1) (aϕ(qi am−ϕ(m) q m−ϕ(m) lm ul Mặt khác, rõ ràng m − ϕ(m) ≥ αi (vì có αi số không nguyên tố với m z at nh oi q1α1 q2α2 qiαi ) Do am−ϕ(m) q m−ϕ(m) qiαi Định lý chứng minh z @ gm Định lý 1.4 (Định lý Fermat) Cho p số nguyên tố a số nguyên m (mod p) co ap−1 ≡ l không chia hết cho P ta có an Lu Chứng minh Theo giả thiết, ta có ϕ(p) = p − a nguyên tố với p nên n va theo định lý Euler ta ap−1 ≡ (mod p) ac th si Định lý 1.5 (Định lý Fermat dạng khác) Cho p số nguyên tố a số nguyên tùy ý ta có ap ≡ a (mod p) Chứng minh Nếu a chia hết cho p hiển nhiên ap ≡ a (mod p) Nếu a khơng chia hết cho p theo định lý 1.4 ta có ap−1 ≡ (mod p) sau nhân hai vế đồng dư thức với a ta ap ≡ a (mod p) Ngược lại từ định lý 1.5 ta suy định lý 1.4 Thật vậy, từ ap ≡ a (mod p) a số nguyên khơng chia hết cho số ngun tố p a nguyên tố với p nên cách chia cho a ta ap−1 ≡ (mod p) Chính vậy, người lu ta nói định lý 1.5 dạng khác định lý Fermat an n va Ví dụ 1.2 Tìm số nguyên x để 9x + tích hai số nguyên liên tiếp tn to Lời giải Giả sử 9x + = n(n + 1) với n ∈ Z ⇒ 36x + 20 = 4n2 + 4n suy ie gh 36x + 21 = (2n + 1)2 ⇒ 3(12x + 7) = (2n + 1)2 p Số phương (2n + 1)2 chia hết chia hết cho Mặt khác nl w (12x + 7) không chia hết 3(12x + 7) không chia hết cho d oa Mâu thuẫn chứng tỏ không tồn số nguyên x để 9x + = n(n + 1) lu Ví dụ 1.3 Tìm số nguyên x để biểu thức sau số phương nf va an x4 + 2x3 + 2x2 + x + Lời giải Đặt x4 + 2x3 + 2x2 + x + = y với y ∈ N lm ul Ta thấy y = (x4 + 2x3 + x2 ) + (x2 + x + 3) = (x2 + x)2 + (x2 + x + 3) x+ + > ⇒ y > a2 z at nh oi Đặt x2 + x = a ta có y = a2 + (x2 + x + 3) Từ có y − a2 = x2 + x + =   11 Dễ thấy =3 x+ + ⇒ (a + 2)2 > y Do a2 < y < (a + 2)2 ⇒ y = (a + 1)2 (a + 2)2 − y2   z @ gm Suy x4 + 2x3 + 2x2 + x + = (x2 + x + 1)2 Suy x2 + x − = ⇒ x = 1; x = −2 co l Thử lại, với x = 1; x = −2 biểu thức = 32 Vậy x = 1; x = −2 giá trị m cần tìm (1.1) n va xy = z an Lu Ví dụ 1.4 Tìm nghiệm ngun dương phương trình ac th si Lời giải Giả sử x0 , y0 , z0 thỏa mãn (1.1) có ƯSCLN d Giả sử x0 = dx1 , y0 = dy1 , z0 = dz1 (x1 , y1 , z1 ) thỏa mãn (1.1) Do đó, ta giả sử (x, y, z) = x, y, z đơi ngun tố hai ba số x, y, z có ước chung d số cịn lại chia hết cho d Ta có x.y = z mà (x, y) = nên x = a2 , y = b2 với a, b ∈ N∗ Bởi (1.1) ⇔ z = x.y = (ab)2 ⇔ z = (ab) Như ta biểu thức nghiệm x = ta2 ; y = tb2 ; z = ab (t ∈ N∗ ) Ngược lại, dễ thấy số x, y, z có dạng thỏa mãn (1.1) Vậy cơng thức cho ta tất nghiệm nguyên dương (1.1) lu an Ví dụ 1.5 Tìm tất nghiệm nguyên phương trình n va (1.2) tn to x2 + xy + y = x2 y ie gh Lời giải (1.2) ⇔ x2 + 2xy + y = x2 y + xy ⇔ (x + y)2 = xy(xy + 1) p Ta thấy xy xy + hai số nguyên liên tiếp nên: nl w + Xét xy = 0, ta có xy = x2 + y = d oa ⇔ x = y = lu nf va an + Xét xy + = 0, ta có : xy = −1 x2 + y = ⇔ (x, y) = (1, −1); (−1, 1) lm ul Thử lại, ba cặp số (0, 0); (1, −1); (−1, 1) thỏa mãn phương trình cho z at nh oi Vậy phương trình có ba nghiệm nguyên (x, y) = (0, 0); (1, −1); (−1, 1) z b Hàm tổng ước số tự nhiên @ ước n co l gm Định nghĩa 1.2 (xem [2],[3]) Cho số nguyên dương n Ta ký hiệu σ(n) tổng m Định lý 1.6 (xem [2],[3]) Hàm số σ(n) có tính chất nhân tính theo nghĩa: Nếu an Lu a, b hai số nguyên tố σ(ab) = σ(a)σ(b) n va ac th si 61 b) Chọn đa thức P (x) = x2 + với x ∈ R, ta có P (x) − P (x) ≡ x2 − 2x + > P (x) − P 00 (x) ≡ x2 + > 0, nghĩa khẳng định khơng cịn Bài toán 3.2 (VMO - 95) Hãy xác định tất đa thức P(x) thỏa mãn điều kiện sau: Với số a > 1995 số nghiệm thực phương trình P (x) = a (mỗi nghiệm tính với số bội nó) bậc đa thức P(x), nghiệm thực phương trình lớn 1995 lu Lời giải Do yêu cầu nghiệm thực P(x) = a lớn 1995 nên an xét đa thức P(x) có bậc n ≥ va n - Xét đa thức P(x) bậc n hàm đơn điệu (−∞; +∞) thỏa mãn đề to Vì đồ thị hàm P(x) có hữu hạn điểm uốn nên với a đủ lớn a > 1995 tn p ie gh P (x) = a có tối đa nghiệm (mỗi nghiệm tính với số bội nó), a−c suy n = P(x) có dạng bx + c với b > 0; nghiệm P(x) x = Ta b có x > 1995 với a > 1995 b > c ≤ 1995(1 − b) w oa nl - Xét đa thức P(x) có hàm số cực trị (−∞; +∞) thỏa mãn đề d n ≥ Giả sử P(x) đạt cực đại m điểm u1 ; u2 ; ; um (m ≥ 1) đạt cực tiểu an lu k điểm v1 ; v2 ; ; vk (k ≥ 1) nf va Đặt d = max {P (u1 ); P (u2 ); ; P (um ); P (v1 ); P (v2 ); ; P (vk )} Do đồ thị hàm P(x) có hữu hạn điểm uốn nên với a đủ lớn a > lm ul max {d, 1995}, suy n = z at nh oi P(x) = a có tối đa hai nghiệm (mỗi nghiệm tính với số bội nó), Nhưng P(x) tam thức bậc hai với a đủ lớn a > 1995 P(x) = a z gm @ có tối đa nghiệm lớn 1995, đa thức lại không thỏa mãn đề Vậy đa thức P(x) thỏa mãn đề có dạng P(x) = bx + c với b > l c ≤ 1995(1 − b) m co x an Lu Bài toán 3.3 (TST 1994) Cho p(x) ∈ R [x] deg p(x) = Giả sử p(x) = có   − 4x − 4x nghiệm dương phân biệt Chứng minh p(x) + − p (x) − 2 x n va ac th si 62 p00 (x) = có nghiệm dương phân biệt Lời giải Ta chứng minh bổ đề sau p(x) = có nghiệm phân biệt < x1 < x2 < x3 < x4 phương trình p(x) − p0 (x) = có nghiệm phân biệt y1 , y2 , y3 , y4 thỏa mãn < x1 < y1 < x2 < y2 < x3 < y3 < x4 < y4 Xét f (x) = e−x p(x) f (x) = ⇔ p(x) = Vậy f (x) = có nghiệm < x1 < x2 < x3 < x4 Áp dụng định lý Lagrange ta có f (x) = −e−x p(x) + p0 (x)e−x f (x) = ⇔ p(x) − p0 (x) = Do phương trình p(x) − p0 (x) = có nghiệm y1 < y2 < y3 deg(p − p0 ) = lu an nên phương trình p(x) − p0 (x) = có nghiệm thứ y4 Không giảm tổng quát n va ta giả sử hệ số x4 p(x) dương to Suy lim (p(x) − p0 (x)) = +∞ nên ∃α > x4 cho p(α) − p0 (α) > x→∞ gh tn Do p(y3 ) − p0 (y3 ) = p(y2 ) − p0 (y2 ) = nên y4 ∈ (β, α) ie Vậy < x1 < y1 < x2 < y2 < x3 < y3 < x4 < y4 p Suy Q(x) := p(x) − p0 (x) = có nghiệm dương phân biệt y1 , y2 , y3 , y4 oa nl w Giả sử Q(x) = ax4 + bx3 + cx2 + dx + e (a, e 6= 0) 1 1 1 Khi R(x) := x Q = có nghiệm dương phân biệt , , , , d lu Vậy nên R(x) − R0 (x) x y1 y2 y3 y4 = có nghiệm dương phân biệt 1 nf va an Ta có R(x) − R0 (x) = x4 Q − 4x3 Q 1 + x2 Q 1 z at nh oi lm ul ⇔ =0 xh    i x h   x  i  i 00 ⇔x p −p − 4x p −p +x p −p =0 x x   x x   x  x 1 + (−x4 + 4x3 + x2 )p0 − x2 p00 =0 ⇔ (x4 − 4x3 )p x  x x     1 ⇔ (x2 − 4x)p + (−x2 + 4x + 1)p0 − p00 =0 x x x   1 − 4t − 4t Đặt t = , phương trình có dạng p(t) + − p (t) − p00 (t) = 0, x t t h 1 z gm @ toán chứng minh l Bài toán 3.4 (VMO - 2012) Cho hai cấp số cộng (an ) , (bn ) với m số nguyên m co dương, m > Xét m tam thức bậc hai pk (x) = x2 + ak x + bk (k = 1, 2, , m) lại khơng có nghiệm thực an Lu Chứng minh p1 (x) pm (x) khơng có nghiệm thực tam thức n va ac th si 63 Lời giải Ta có tam thức bậc haip1 (x) pm (x) khơng có nghiệm thực suy p1 (x) > ∀x ∈ R pm (x) > ∀x ∈ R Giả sử tồn pk (x) = x2 + ak x + bk (k = 2, , m − 1) có nghiệm thực x = c Khi pm (x) − pk (x) = (m − k) (ax + b) pk (x) − p1 (x) = (k − 1) (ax + b) (ở a, b công sai hai cấp số cộng(an ) , (bn )) Do pm (c) = (m − k) (ac + b) p1 (c) = − (k − 1) (ac + b) nên pm (c) p1 (c) < 0, vô lý Vậy tam thức bậc hai cịn lại khơng có nghiệm thực lu Bài toán 3.5 (IMO Shortlist 2006) Cho P (x) đa thức với hệ số nguyên có an bậc n > k số nguyên dương va n Xét đa thức Q (x) = P k (x), với P (x) tác động k lần Chứng minh gh tn to có nhiều n số nguyên t cho Q (t) = t p ie Lời giải Trước hết ta chứng minh bổ đề: Nếu t số nguyên thỏa mãn Q (t) = t w P (t) = t oa nl Thật ta có d (P (t) − t)|(P (t) − P (t))| |(P k (t) − P k−1 (t))|(P k+1 (t) − P k (t)) an lu nf va Mặt khác, P k+1 (t) − P k (t) = P (t) − t z at nh oi lm ul nên |P (t) − t| = P (t) − P (t) = · · · = P k (t) − P k−1 (t) , z Đặt d = P (t) − t Nếu d = P (t) = t, suy P (t) = P (t) = t   Nếu d 6= 0, giả sử i số nhỏ mà d = − P i (t) − P i−1 (t) , ≤ i ≤ k @ gm Khi an Lu Ngược lại m Suy P i (t) = P i−2 (t) nên P (t) = t co l P i−1 (t) − P i−2 (t) = P i−1 (t) − P i (t) n va d = P (t) − t = P (t) − P (t) = · · · = P k (t) − P k−1 (t) ac th si 64 P k (t) = t + kd 6= t, mâu thuẫn Trở lại tốn, giả sử có (n + 1) số ngun t1 < t2 < · · · < tn < tn+1 thỏa mãn Q (ti ) = ti , ≤ i ≤ n + Khi theo bổ đề nêu trên, ta có P (ti ) = ti , ≤ i ≤ n + Với ≤ i < j ≤ n + 1, ta có ti − tj |P (ti ) − P (tj ) P (ti ) − P (tj ) nên |P (ti ) − P (tj )| = tj − ti Theo bất đẳng thức giá trị tuyệt đối, ta có lu tn+1 − t1 = |P (tn+1 ) − P (t1 )| an n va ≤ |P (tn+1 ) − P (tn )| + |P (tn ) − P (tn−1 )| + · · · + |P (t2 ) − P (t1 )| = tn+1 − t1 Giả sử tất hiệu P (ti ) − P (tj ) dấu dương, ie gh tn to Do đó, tất hiệu P (ti+1 ) − P (ti ) dấu p P (ti+1 ) − P (ti ) = ti+1 − ti , ∀1 ≤ i ≤ n, w oa nl suy P (ti+1 ) − ti+1 = P (ti ) − ti , ∀1 ≤ i ≤ n Do đa thức P (x) − x − (P (ti ) − ti ) , có n + nghiệm phân biệt t1 < t2 < d an lu · · · < tn < tn+1 , điều vơ lý đa thức P (x) − x − (P (ti ) − ti ) , đa thức bậc n nf va Tương tự cho trường hợp tất hiệu P (ti+1 ) − P (ti ) đề dấu âm z at nh oi lm ul Vậy ta có (đpcm) Các đề toán toán rời rạc liên quan 3.2.1 Một số toán cực trị tập số nguyên z 3.2 gm @ x4 y (x + yz)(y + zx)(z + xy)3 an Lu Tìm giá trị lớn M = m z(z − x − y) = x + y + co l Bài toán 3.6 Cho x, y, z số nguyên dương thỏa mãn điều kiện n va ac th si 65 Lời giải Ta chứng minh bất đẳng thức x4 y 36 ≤ (x + yz)(y + zx)(z + xy)3 Vì z(z − x − y) = x + y + → (z + 1)(x + y) = z − z > nên x + y + = z Khi P = x4 y x4 y = (x + y)(1 + y)(x + y)(1 + x)[(x + 1)(y + 1)]3 (x + y)2 [(x + 1)(y + 1)]4 Áp dụng bất đẳng thức AM-GM cho số dương x, y, ta có r x x x   x3  4 4x (x + 1) = lu an (y + 1) = + + y ≥ +1 y y + + +1 3 4  r ≥ =4 27 y3 27 4 27 = 44 y3 27 n va (x + y)2 ≥ 4xy tn to Do (x + y)2 [(x + 1)(y + 1)]4 ≥ 4xy.48 p ie gh Dấu đẳng thức xảy  x = y = 3 z = x + y + 49 4 36 x3 y = x y suy P ≤ 36 36 49 36 49 oa nl w ⇔ x = 3, y = 3, z = d Vậy max M = lu nf va an Bài toán 3.7 Cho a, b, c nguyên dương a + b + c = 3n + (n nguyên dương cho trước) Tìm giá trị lớn T = abc Ta chứng minh bất đẳng thức abc ≤ (n)2 (n + 1) Ta coi lm ul Lời giải a ≥ b ≥ c Khi a ≥ (n + 1) z at nh oi Ta có p na + (n + 1)b + (n + 1)c (Theo AM-GM) p (n + 1)(a + b + c) − a ⇔ (na)(n + 1)b(n + 1)c ≤ p (n + 1)(3n + 1) − (n + 1) ⇔ (na)(n + 1)b(n + 1)c ≤ (n(n + 1))3 ⇔ abc ≤ n(n + 1)2 (na)(n + 1)b(n + 1)c ≤ z m co l gm @ an Lu ⇔ abc ≤ n2 (n + 1) n va Vậy abc ≤ n2 (n + 1) ⇔ a = n + 1, b = n, c = n Suy max T = n2 (n + 1) ac th si

Ngày đăng: 24/07/2023, 09:00

Tài liệu cùng người dùng

Tài liệu liên quan